LSAT and Law School Admissions Forum

Get expert LSAT preparation and law school admissions advice from PowerScore Test Preparation.

User avatar
 Dave Killoran
PowerScore Staff
  • PowerScore Staff
  • Posts: 5850
  • Joined: Mar 25, 2011
|
#80543
This game is also discussed in our Podcast: LSAT Podcast Episode 70: The May 2020 LSAT-Flex Logic Games Section

Complete Question Explanation
(The complete setup for this game can be found here: https://forum.powerscore.com/lsat/viewtopic.php?t=33051)

The correct answer choice is (B).


If the only meeting M hosts is in the spring, then via the third rule V must host a meeting in the fall:

  • ..... ..... ____ ..... ..... ..... ____

    ..... ..... ____ ..... ..... ..... ____

    ..... ..... _V_ ..... ..... ..... _M_
    ..... ..... Fall ..... ..... ..... Spring
Of course, if M hosts only one meeting and it is in the spring, then M cannot hosts a meeting in the fall, which enacts the contrapositive of the first rule. This means that H also cannot host a meeting in the fall. With H and F removed as host options for the fall, we can then determine that O and T must host meeting in the fall (since there were only 5 variables to fill 3 spaces and 2 have been eliminated):

  • ..... ..... _T_ ..... ..... ..... ____

    ..... ..... _O_ ..... ..... ..... ____

    ..... ..... _V_ ..... ..... ..... _M_
    ..... ..... Fall ..... ..... ..... Spring
    ..... ..... M
    ..... ..... H
Consequently, since H cannot host a meeting in the fall and H must host at least one meeting, H must host a meeting in the spring:

  • ..... ..... _T_ ..... ..... ..... ____

    ..... ..... _O_ ..... ..... ..... _H_

    ..... ..... _V_ ..... ..... ..... _M_
    ..... ..... Fall ..... ..... ..... Spring
    ..... ..... M
    ..... ..... H
With only a single space remaining in the spring semester, and neither of the spring host cities being T, we can conclude that V cannot host a meeting in the spring as there would not be enough spaces to fulfill the requirements of the second rule. Thus, the remaining choice is whether O or T host the final meeting in the spring:

  • ..... ..... _T_ ..... ..... ..... _O/T_

    ..... ..... _O_ ..... ..... ..... _H_

    ..... ..... _V_ ..... ..... ..... _M_
    ..... ..... Fall ..... ..... ..... Spring
    ..... ..... M ..... ..... .....       V
    ..... ..... H


Answer choice (A): Because M cannot host a meeting in the fall, from the contrapositive of the first rule we know H cannot host a meeting in the fall. Thus, H cannot host meetings in both semesters and this answer choice is incorrect.

Answer choice (B): This is the correct answer choice. As shown above T could host a meeting in both semesters.

Answer choice (C): Because M cannot host a meeting in the fall, from the contrapositive of the first rule we know H cannot host a meeting in the fall. Thus, this answer choice is incorrect.

Answer choice (D): If O does not host a meeting in the fall, then there will not be three different cities available to fill the three fall spaces. Thus, this answer choice cannot occur and is incorrect.

Answer choice (E): If T does not host a meeting in the fall, then there will not be three different cities available to fill the three fall spaces. Thus, this answer choice cannot occur and is incorrect.

Get the most out of your LSAT Prep Plus subscription.

Analyze and track your performance with our Testing and Analytics Package.